How to Obtain Fundamental Solutions for Non-Constant Coefficient Equations?

Click For Summary
To solve the non-constant coefficient equation using the method of variation of parameters, one must first address the corresponding homogeneous equation, which is y'' - (4/x)y' + (6/x^2)y = 0. The key to finding fundamental solutions is to assume a solution of the form y(x) = x^r and substitute it into the homogeneous equation to determine the values of r that satisfy it. This approach can be simplified using the Frobenius method, which also provides a systematic way to find solutions for such equations. Understanding the units of each term in the equation is crucial, as it helps in recognizing the form of potential solutions. Ultimately, finding the fundamental solutions allows for the application of variation of parameters to solve the original non-homogeneous equation.
cheeee
Messages
15
Reaction score
0
Solve by method of variation of parameters
(x^2)y'' - (4x)y' + 6y = x^4*sinx (x > 0)

Hey, I know how to solve problems using variation of parameters but only when the corresponding homogenous equation has constant coefficients...

y'' - (4/x)y' + (6/x^2)y = 0.. the bit I am confused about is how to obtain the fundamental solutions to this equation {y1, y2} when the coefficients are not constants. Any help would be appreciated.

Thanks.
 
Physics news on Phys.org
Note that each term has the same "units", if you think of x as having units of length (so that a derivative removes one unit of length). Such differential equations have solutions of the form x^r. Plug this in and solve for r, and you'll quickly see why such solutions work. This is something you can just remember, although it would also fall out if you tried the Frobenius method.
 
im not quite sure I understand where to plug in x^r .
 
I mean y(x)=xr is a solution to the homogenous differential equation for certain r. Plug in this y and see which r work.
 
Question: A clock's minute hand has length 4 and its hour hand has length 3. What is the distance between the tips at the moment when it is increasing most rapidly?(Putnam Exam Question) Answer: Making assumption that both the hands moves at constant angular velocities, the answer is ## \sqrt{7} .## But don't you think this assumption is somewhat doubtful and wrong?

Similar threads

  • · Replies 7 ·
Replies
7
Views
2K
  • · Replies 3 ·
Replies
3
Views
2K
Replies
2
Views
2K
  • · Replies 1 ·
Replies
1
Views
2K
  • · Replies 11 ·
Replies
11
Views
2K
  • · Replies 2 ·
Replies
2
Views
918
Replies
7
Views
2K
  • · Replies 3 ·
Replies
3
Views
2K
Replies
6
Views
2K
  • · Replies 2 ·
Replies
2
Views
1K